Official Explanation
3. The author would most likely agree with which of the following?
Difficulty Level: 650
Analyze the Question Stem:
The phrase "would most likely agree" means that this is an Inference question asking for the author's opinion.
Research the Relevant Text:
Since this question does not specify a particular topic in the passage, the correct answer could be based on any part of it and there's no way to research now. Instead, evaluate the choices and research them as needed.
Make a Prediction:
It is difficult to make a prediction on open-ended Inference questions. The best plan is to move straight to the choices and evaluate them in turn, keeping the Scope and Purpose in mind.
Evaluate the Answer Choices:
(A) is incorrect because the author never expresses any opinion about whether rent control should be used. She only discusses its effects (both positive and negative) in the cases where it is used.
Since the author states in the first paragraph that rent controls represent an upper limit on rent increases, controls would not restrict a landlord from charging the market-demanded increase in a year in which that increase is less than the limit imposed by rent control. For example, if the rent control law limits increase to 5% per year, and if the market demands only a 2% increase in a particular year, a landlord could charge the entire 2% without running afoul of the rent control policy. Thus, in some years rent controls might not affect rent increases at all, making (B) is correct.
(C) is incorrect because the author does not provide any means of comparing the costs of rent control to its benefits.
(D) is an irrelevant comparison; the author does not express any opinion on the desirability of living in any city, rent-controlled or otherwise.
(E) may be true, but there's no way to infer this statement from the contents of the passage.
Answer: B
TAKEAWAY: Keep in mind on Inference questions that you must be able to cite a specific part of the passage that supports the answer you choose.
Naila02
Could you please explain 3rd one?
Thank you in advance
Sajjad1994phuongtth4
Could you pls explain the 3rd question. I chose C

Thank you in advance.